Đến nội dung

Hình ảnh

[TOPIC] Sáu Bảy Tám Chín.


  • Please log in to reply
Chủ đề này có 105 trả lời

#81
tritanngo99

tritanngo99

    Đại úy

  • Điều hành viên THPT
  • 1644 Bài viết

Bài 141: Chứng tỏ rằng nếu $F_n=2^{2^{n}}+1$ với $n>1$ là một số nguyên tố thì $2$ không phải là căn nguyên thủy của $F_n$.

Bài 142: Trong phòng có $6$ người. Chứng minh rằng tồn tại $3$ người đôi một quen nhau hoặc ba người đôi một không quen nhau.

Bài 143: Cho đường tròn $(O;R)$ và điểm $M$ cố định, $OM=d$. Một đường thẳng thay đổi qua $M$ cắt đường tròn tại hai điểm $A$ và $B$. Khi đó: $\overline{MA}.\overline{MB}=MO^2-R^2=d^2-R^2$.

Bài 144: [Định lý Menelauyt] Cho tam giác $ABC$, cho các điểm $D,E,F$ theo thứ tự nằm trên các đường thẳng $BC,CA,AB$. Khi đó $D,E,F$ thẳng hàng khi và chỉ khi $\frac{\overline{FA}}{\overline{FB}}.\frac{\overline{DB}}{\overline{DC}}.\frac{\overline{EC}}{\overline{EA}}=1$


Bài viết đã được chỉnh sửa nội dung bởi tritanngo99: 08-12-2018 - 21:38


#82
tritanngo99

tritanngo99

    Đại úy

  • Điều hành viên THPT
  • 1644 Bài viết

Lời giải bài 141: Từ sự phân tích: $2^{2^{n+1}}-1=(2^{2^{n}}-1)(2^{2^{n}}+1)$, ta có: $2^{2^{n+1}}\equiv 1(\text{ mod }F_n)$. Từ đó suy ra, cấp của $2$ theo modulo $F_n$ không vượt quá $2^{n+1}$. Nhưng với giả thiết $F_n$ là số nguyên tố thì $\phi(F_n)=F_n-1=2^{2^{n}}$.

Bằng quy nạp đơn giản dễ dàng nhận được $2^{2^{n}}>2^{n+1},\forall n>1$. Do vậy cấp của $2$ theo modulo $F_n$ nhỏ hơn $\phi(F_n)$. Do đó $2$ không thể là căn nguyên thủy của $F_n$

Lời giải bài 142: Xét $6$ điểm trên mặt phẳng. Chọn $1$ điểm bất kì ta dùng đoạn nối liền giữa các điểm thể hiện sự quen nhau và các điểm nối nét đứt với nhau chỉ sự không quen nhau.

Bây giờ ta xét $6$ điểm $O,A,B,C,D,E$ lấy $O$ là gốc.

Trong $5$ điểm còn lại, ta thấy $2$ người bất kì hoặc là quen nhau, hoặc là không quen nhau, trên hình theo nguyên lý Dirichlet thì tồn tại ít nhất $3$ đoạn thẳng nét liền từ $O$ đến $5$ điểm $A,B,C,D,E$ hoặc $3$ đoạn nối nét đứt.

Bây giờ ta chỉ cần xét sự quen nhau, thật vậy nếu trong $3$ điểm $A,B,C$ mà nối lại với nhau thì ta được $1$ tam giác có đỉnh là $O$ suy ra thỏa mãn bài toán, nếu không nối lại thì $3$ điểm $A,B,C$ sẽ chỉ nối nhau bằng nét đứt cũng là điều phải chứng minh. Vậy ta có điều phải chứng minh.  

Lời giải bài 143: Gọi $C$ là điểm đối xứng của $A$ qua $O$. Ta có $CB\bot AM$ hay $B$ là hình chiếu của $C$ trên $AM$.

Khi đó ta có: $\overline{MA}.\overline{MB}=\vec{MA}.\vec{MB}=\vec{MC}.\vec{MA}=(\vec{MO}+\vec{OC})(\vec{MO}+\vec{OA})=(\vec{MO}-\vec{OA})(\vec{MO}+\vec{OA})=\overline{MO}^2-\overline{OA}^2=OM^2-OA^2=d^2-R^2$ 

Lời giải bài 144: 

Thuận: Giả sử $D,E,F$ thẳng hàng. Từ $C$, kẻ $CI\parallel AB(I \in DF)$, áp dụng định lý Ta-lét, ta có:

$\frac{\overline{EC}}{\overline{EA}}=\frac{\overline{IC}}{\overline{FA}};\frac{\overline{DB}}{\overline{DC}}=\frac{\overline{FB}}{\overline{IC}}\implies \frac{\overline{FA}}{\overline{FB}}.\frac{\overline{DB}}{\overline{DC}}.\frac{\overline{EC}}{\overline{EA}}=1$.

Đảo:

Giả sử các điểm $D,E,F$ thỏa mãn $\frac{\overline{FA}}{\overline{FB}}.\frac{\overline{DB}}{\overline{DC}}.\frac{\overline{EC}}{\overline{EA}}=1$, lấy $F'\in AB$ sao cho $D,E,F'$ thẳng hàng. Theo chiều thuận định lý Melelaus, ta có: $\frac{\overline{F'A}}{\overline{FB}}.\frac{\overline{DB}}{\overline{DC}}.\frac{\overline{EC}}{\overline{EA}}=1,$ suy ra $\frac{\overline{F'A}}{\overline{FB}}=\frac{\overline{FA}}{\overline{FB}}$ hay $2$ điểm $F'$ và $F$ cùng chia đoạn $AB$ theo cùng tỉ số. Vậy $F\equiv F'$ hay $D,E,F$ thẳng hàng.


Bài viết đã được chỉnh sửa nội dung bởi tritanngo99: 02-12-2018 - 17:53


#83
tritanngo99

tritanngo99

    Đại úy

  • Điều hành viên THPT
  • 1644 Bài viết

Bài 145: Thực hiện trò chơi sau: Lần đầu, viết lên bảng cặp số $(2,\sqrt{2})$. Từ lần thứ $2$, nếu trên bảng có cặp số $B=(a,b)$ thì được phép viết thêm cặp số $T(B)=(\frac{a+b}{2},\frac{a-b}{2})$. Hỏi ta có thể viết được lên bảng cặp số $(1,1+\sqrt{2})$ hay không?

Bài 146: Số $2012!$ có tận cùng bao nhiêu chữ số $0$.

Bài 147: Cho tam giác $ABC,$ các điểm $M,N,P$ lần lượt nằm trên các đường thẳng $BC,CA,AB$. Khi đó $\frac{S_{[MNP]}}{S_{[ABC]}}=\frac{\overline{BM}.\overline{CN}.\overline{AP}-\overline{CM}.\overline{AN}.\overline{BP}}{\overline{AB}.\overline{BC}.\overline{CA}}$.

Bài 148: Cho tứ giác $ABCD$ và đường thẳng $d$ cắt $AB,BC,CD,DA$ lần lượt ở $M,N,P,Q$. Khi đó: $\frac{\overline{MA}}{\overline{MB}}.\frac{\overline{NB}}{\overline{NC}}.\frac{\overline{PC}}{\overline{PD}}.\frac{\overline{QD}}{\overline{QA}}=1$.



#84
tritanngo99

tritanngo99

    Đại úy

  • Điều hành viên THPT
  • 1644 Bài viết

Lời giải bài 145: Giả sử ở bước thứ $n$ ta viết cặp số $(a_n,b_n)$. Xét $S(n)=a_n^2+b_n^2$, ta có: $S(n+1)=(\frac{a_n+b_n}{2})^2+(\frac{a_n-b_n}{2})^2=S(n),\forall n\ge 1$.

Do đó $S(n)$ là bất biến trong trò chơi nói trên.

Mà $S(1)=6$ và $1+(1+\sqrt{2})^2\ne 6$ nên ta không thể viết được cặp số $(1,1+\sqrt{2})$.

Lời giải bài 146:

Bổ đề: Cho $a>0$ và số $n$ nguyên dương. Chứng minh rằng số các số nguyên dương là bội số của $n$ và không vượt quá $a$ là $[\frac{a}{n}]$. 

Chứng minh bổ đề:

Ta viết: $a=nq+r$, trong đó $q$ là số tự nhiên, $0\le r<n$.

Rõ ràng các bội số của $n$ không vượt quá $a$ là $n,2n,...,qn$, tổng cộng có $q$ số. Mặt khác: $[\frac{a}{n}]=q$. Từ đó suy ra kết luận của bài toán.

Vì $10=2.5$ nên để biết $2012!$ có tận cùng bằng bao nhiêu chữ số $0$, ta cần phải tính số mũ của $5$ khi phân tích $2012!$ ra thừa số nguyên tố.

Theo bổ đề trên, số mũ của $5$ khi phân tích $2012!$ ra thừa số nguyên tố bằng: $[\frac{2012}{5}]+[\frac{2012}{5^2}]+[\frac{2012}{5^3}]+[\frac{2012}{5^4}]=402+80+16+3=501.$( Do $2012<5^5$).

Số mũ của $2$ khi phân tích $2012!$ ra thừa số nguyên tố nhiều hơn $501$.

Vậy $2012!$ có tận cùng là $501$ chữ số $0$.

Nhận xét: Nếu $5^k\le n<5^{k+1}$ thì số chữ số $0$ tận cùng về bên phải của số $n!$ bằng: $[\frac{n}{5}]+[\frac{n}{5^2}]+...+[\frac{n}{5^k}]$.

Lời giải bài 147: Ta có $S_{[ABC]}=S_{[MAB]}+S_{[MCA]}=S_{[PMA]}+S_{[PBM]}+S_{[NMC]}+S_{[NAM]}=S_{[MNP]}+S_{[BMP]}+S_{[CNM]}+S_{[APN]}$.

Mặt khác: $\frac{S_{[BMP]}}{S_{[ABC]}}=\frac{\overline{BM}.\overline{BP}}{\overline{BC}.\overline{BA}};\frac{S_{[CNM]}}{S_{[ABC]}}=\frac{\overline{CN}.\overline{CM}}{\overline{CA}.\overline{CB}};\frac{S_{[APN]}}{S_{[ABC]}}=\frac{\overline{AP}.\overline{AN}}{\overline{AB}.\overline{AC}}$.

Suy ra: $\frac{S_{[MNP]}}{S_{[ABC]}}=1-\frac{S_{[BMP]}}{S_{[ABC]}}-\frac{S_{[CNM]}}{S_{[ABC]}}=\frac{\overline{BM}\overline{CN}.\overline{AP}-\overline{CM}.\overline{AN}.\overline{BP}}{\overline{AB}.\overline{BC}.\overline{CA}}$(đpcm).

Lời giải bài 148: Trên $d$ lấy $I,J$ sao cho $AI\parallel BJ\parallel CD$.

Theo định lý Thales, ta có: $\frac{\overline{MA}}{\overline{MB}}=\frac{\overline{IA}}{\overline{JB}};\frac{\overline{NB}}{\overline{NC}}=\frac{\overline{JB}}{\overline{PC}};\frac{\overline{QD}}{\overline{QA}}=\frac{\overline{PD}}{\overline{IA}}$. Từ đó suy ra đpcm.

Chú ý: Dạng đảo của định lý trên không đúng và định lý trên có thể mở rộng cho đa giác bất kỳ.

 

Bài viết đã được chỉnh sửa nội dung bởi tritanngo99: 05-12-2018 - 18:33


#85
tritanngo99

tritanngo99

    Đại úy

  • Điều hành viên THPT
  • 1644 Bài viết

Bài 149: Tìm số tự nhiên $k$ lớn nhất sao cho $(2011!)^{2012}$ chia hết cho $2012^k$.

Bài 150: Bàn cờ vua $8x8$ bị mất hai ô ở hai góc đối diện. Hỏi có thể lát phần còn lại của bàn cờ bởi các quân Domino $2x1$ được không?

Bài 151: [Định lý Ceva]Cho tam giác $ABC$, các điểm $E,F,G$ tương ứng nằm trên $BC,CA,AB$. Ba đường thẳng $AE,BF,CG$ đồng quy tại một điểm $O$ khi và chỉ khi $\frac{\overline{GA}}{\overline{GB}}.\frac{\overline{EB}}{\overline{EC}}.\frac{\overline{FC}}{\overline{FA}}=-1$.

Bài 152: [Định lý Ceva dạng lượng giác (Ceva sin)]Cho tam giác $ABC$, các điểm $E,F,G$ tương ứng nằm trên $BC,CA,AB$. Ba đường thẳng $AE,BF,CG$ đồng quy tại một điểm $O$ khi và chỉ khi $\frac{sin(\angle{ABF})}{sin(\angle{CBF})}.\frac{sin(\angle{BCG})}{sin(\angle{ACG})}.\frac{sin(\angle{CAE})}{sin(\angle{BAE})}=1$.


Bài viết đã được chỉnh sửa nội dung bởi tritanngo99: 08-12-2018 - 21:38


#86
tritanngo99

tritanngo99

    Đại úy

  • Điều hành viên THPT
  • 1644 Bài viết

Lời giải bài 149: Ta có: $2012=2^2.503$.

Số mũ cao nhất của $503$ có trong $2011!$ là: $[\frac{2011}{503}]=3$( do $2011<503^2$).

Vậy $2011!$ chia hết cho $503^3$ và không chia hết cho $503^4$, hiển nhiên $2011!$ chia hết cho $4^3$. Do vậy $2011!$ chia hết cho $2012^3$ và không chia hết cho $2012^4$.

Muốn cho $(2011!)^{2012}$ chia hết cho $2012^k$ thì $k\le 3.2012=6036$.

Vậy $Max(k)=6036$.

Lời giải bài 150: Mỗi quân Domino lát vào bàn cờ luôn chiếm một ô trắng và một ô đen. Do đó, nếu lát được phần còn lại của bàn cờ thì số ô trắng và số ô đen bằng nhau. Nhưng do hai ô ở hai góc đối diện của bàn cờ là hai ô vuông cùng màu nên số ô màu trắng và số ô màu đen trong phần còn lại của bàn cờ không bằng nhau. Vậy không lát được phần còn lại của bàn cờ bằng các quân Domino.

bc.jpg

Lời giải bài 151: 

Thuận: Giả sử ba đường thẳng $AE,BF,CG$ đồng quy tại một điểm $O$. Từ $A$ và $C$, kẻ các đường thẳng song song với $BF$, lần lượt cắt $CG$ và $AE$ tại $K,I$ tương ứng.

Áp dụng định lý Thales, ta có: $\frac{\overline{CF}}{\overline{FA}}=\frac{\overline{CO}}{\overline{OK}};\frac{\overline{CI}}{\overline{AK}}=\frac{\overline{CO}}{\overline{OK}}\implies \frac{\overline{CF}}{\overline{FA}}=\frac{\overline{CI}}{\overline{AK}}$.

Các cặp tam giác đồng dạng $IEC$ và $OEB$, $AKG$ và $BOG$: $\frac{\overline{BE}}{\overline{CE}}=\frac{\overline{BO}}{\overline{CI}};\frac{\overline{AG}}{\overline{BG}}=\frac{\overline{AK}}{\overline{BO}}$.

Do đó: $\frac{\overline{GA}}{\overline{GB}}.\frac{\overline{EB}}{\overline{EC}}.\frac{\overline{FC}}{\overline{FA}}=\frac{\overline{AK}}{\overline{BO}}.\frac{\overline{BO}}{\overline{CI}}.\frac{-\overline{CI}}{\overline{AK}}=-1$(đpcm).

Đảo: Chứng minh tương tự phần đảo bài 144.

Lời giải bài 152: Ta có: $\frac{BE}{CE}=\frac{S_{ABE}}{S_{ACE}}=\frac{AB.sin\angle{BAE}}{AC.sin\angle{CAE}};\frac{CF}{AF}=\frac{BC.sin\angle{CBF}}{BA.sin\angle{ABF}};\frac{AG}{BG}=\frac{CA.sin\angle{ACG}}{CB.\angle{BCG}}$. Nhân theo vế $3$ đẳng thức trên, ta có đpcm.

 

Bài viết đã được chỉnh sửa nội dung bởi tritanngo99: 06-12-2018 - 18:21


#87
tritanngo99

tritanngo99

    Đại úy

  • Điều hành viên THPT
  • 1644 Bài viết

Bài 153: Tìm số tự nhiên $n$ sao cho $[\frac{n}{2010}]=[\frac{n}{2011}]=[\frac{n}{2012}]$.

Bài 154: Xác định các số nguyên dương $m,n$ sao cho bảng $mxn$ có thể lát được bởi các quân hình chữ $L$ dưới đây:

cb.jpg

Bài 155: [Định lý Desargues]Cho $2$ tam giác $ABC$ và $MNP$ có $AM,BN,CP$ đồng quy tại $O$. Gọi $I,J,K$ theo thứ tự là giao điểm của các cặp đường thẳng $(AB,MN),(BC,NP),(CA,PM)$. Khi đó $3$ điểm $I,J,K$ thẳng hàng.

Bài 156: [Định lý Pappus]Cho $2$ đường thẳng $a,b$. Trên $a$ lấy các điểm $A,B,C$; trên $b$ lấy các điểm $X,Y,Z$. Gọi $M$ là giao điểm của $AX$ và $BY$, $N$ là giao điểm của $AZ$ và $CX$, $P$ là giao điểm của $BZ$ và $CY$. Khi đó $M,N,P$ thẳng hàng.  


Bài viết đã được chỉnh sửa nội dung bởi tritanngo99: 08-12-2018 - 21:56


#88
tritanngo99

tritanngo99

    Đại úy

  • Điều hành viên THPT
  • 1644 Bài viết

Lời giải bài 153: Viết $n=2010k+r(0\le r\le 2009,k,r\in \mathbb{N})$. Thay vào biểu thức đề cho ta có:

$[\frac{2010k+r}{2010}]=[\frac{2010k+r-k}{2011}]=[\frac{2012k+r-2k}{2012}]$.

$\iff k=k+[\frac{r-k}{2011}]=k+[\frac{r-2k}{2012}]\iff [\frac{r-k}{2011}]=[\frac{r-2k}{2012}]=0$.

Suy ra: $0\le r-2k$ nên $2k\le r\le 2009,0\le k\le 1004$.

Vậy $n=2010k+r(0\le k\le 1004;2k\le r\le 2009).$

Do có $1005$ giá trị của $k$( từ $0$ đến $1004$). Với mỗi $k$ thì $r$ nhận các giá trị từ $2k$ đến $2009$. Vậy số nghiệm tự nhiên $n$ của $(1)$ là: $\sum\limits_{k=0}^{1004}(2010-2k)=1011030$.

Lời giải bài 154: Không mất tính tổng quát, ta có thể giả sử $m\le n$. Để lát được bảng thì $m\ge 2,n\ge 3$. Giả sử ta có thể lát được bảng bởi $a$ quân hình chữ $L$, ta có: $m*n=4*a$. Vì $m*n\ge 6$ nên $a\ge 2$. Xét $a=2$, ta có bảng $2x4$. Bảng $2x4$ có thể lát được bởi hai quân hình chữ $L$ như hình vẽ dưới đây.

1.jpg

Với $a=3$ ta có $mxn=12$ nên ta có hai bảng thỏa mãn là $2x6$ và $3x4$. Dễ dàng kiểm tra cả hai bảng này đều không lát được bởi các quân hình chữ $L$.

Điều đó khiến ta dự đoán, để lát được bảng bởi các quân hình chữ $L$ thì $a$ chẵn. Để chứng minh dự đoán này ta tô màu các ô của bảng như sau:

Giả sử được $m$ chẵn. Các ô ở dòng có thứ tự lẻ( tính từ trên xuống dưới) được tô màu đen, các ô ở dòng có thứ tự chẵn màu trắng.

2.jpg

Khi đó, số ô đen và số ô trắng bằng nhau và bằng $2a$. Mỗi quân hình chữ $L$ lát vào bảng chiếm $3$ ô đen và $1$ ô trắng hoặc chiếm $3$ ô trắng và $1$ ô đen. Giả sử lát được $x$ quân hình chữ $L$ chiếm $3$ ô đen và $1$ ô trắng và $y$ quân hình chữ $L$ chiếm $3$ ô trắng và $1$ ô đen. Ta có hệ: $\left\{\begin{array}{I} x+y=a\\ 3x+y=3y+x=2a\end{array}\right.$

Suy ra $x=y$ và $a=2x$. Điều đó có nghĩa là $a$ chẵn.

Bây giờ ta chứng minh nếu $a$ chẵn, tức là $mxn$ chia hết cho $8$ thì có thể lát được bảng bởi các quân hình chữ $L$. Thật vậy, nếu $m$ chia hết cho $2$ và $n$ chia hết cho $4$ hình bảng có thể chia thành các hình chữ nhật $2x4$ nên lát được.

Nếu $m$ lẻ và $n$ chia hết cho $8$ thì do $m$ có thể viết được dưới dạng $m=2s+3$ nên có thể chia bảng đã cho thành các hình chữ nhật $2x4$ và $3x8$. Do đó, nếu hình chữ nhật $3x8$ lát được thì bảng đã cho sẽ lát được. Hình vẽ dưới đây chứng tỏ có thể lát được hình chữ nhật này.

3.jpg

Vậy để lát được bảng đã cho bởi các quân hình chữ $L$ thì điều kiện cần và đủ là $mxn$ chia hết cho $8$ và $m,n\ge 2$.

Lời giải bài 155: Áp dụng định lý Menelaus cho các tam giác $OAB,OBC,OCA$ ta có:

$\frac{\overline{IA}}{\overline{IB}}.\frac{\overline{NB}}{\overline{NO}}.\frac{\overline{MO}}{\overline{MA}}=1;\frac{\overline{JB}}{\overline{JC}}.\frac{\overline{PC}}{\overline{PO}}.\frac{\overline{NO}}{\overline{NB}}=1;\frac{\overline{KC}}{\overline{KA}}.\frac{\overline{MA}}{\overline{MO}}.\frac{\overline{PO}}{\overline{PC}}=1$.

Nhân theo vế $3$ đẳng thức trên, ta có: $\frac{\overline{IA}}{\overline{IB}}.\frac{\overline{JB}}{\overline{JC}}.\frac{\overline{KC}}{\overline{KA}}=1$. Suy ra $I,J,K$ thẳng hàng (đpcm).

Định lý đảo của định lý Desargues được phát biểu như sau: Cho hai tam giác $ABC$ và $MNP$ có $AB\cap MN=I,BC\cap NP=J,CA\cap PM=K$ và $I,J,K$ thẳng hàng. Khi đó $AM,BN,CP$ đồng quy tại $O$.

Chứng minh:

Gọi $O$ là giao điểm của $AM$ và $CP$. Áp dụng định lý Menelaus cho các tam giác $CPK,PKJ,JKC$, ta có: $\frac{\overline{OC}}{\overline{OP}}.\frac{\overline{MP}}{\overline{MK}}.\frac{\overline{AK}}{\overline{AC}}=1;\frac{\overline{NP}}{\overline{NJ}}.\frac{\overline{IJ}}{\overline{IK}}.\frac{\overline{MK}}{\overline{MP}}=1;\frac{\overline{BJ}}{\overline{BC}}.\frac{\overline{AC}}{\overline{AK}}.\frac{\overline{IK}}{\overline{IJ}}=1$.

Nhân theo vế $3$ đẳng thức trên, ta có: $\frac{\overline{OC}}{\overline{OP}}.\frac{\overline{NP}}{\overline{NJ}}.\frac{\overline{BJ}}{\overline{BC}}=1\implies O,N,B$ thẳng hàng suy ra $AM,BN,CP$ đồng quy tại $O$(đpcm).

Lời giải bài 156: Gọi $D,E,F$ là giao điểm của các cặp đường thẳng $(AZ,CY),(AZ,BX),(BX,CY)$. Áp dụng Menelaus cho tam giác $DEF$ với cát tuyến $CNX$, ta có: $\frac{\overline{ND}}{\overline{NE}}.\frac{\overline{XE}}{\overline{XF}}.\frac{\overline{CF}}{\overline{CD}}=1\implies \frac{\overline{ND}}{\overline{NE}}=\frac{\overline{CD}}{\overline{CF}}.\frac{\overline{XF}}{\overline{XE}}$.

Tương tự ta có: $\frac{\overline{PF}}{\overline{PD}}=\frac{\overline{ZE}}{\overline{ZD}}.\frac{\overline{BF}}{\overline{BE}},\frac{\overline{ME}}{\overline{MF}}=\frac{\overline{AE}}{\overline{AD}}.\frac{\overline{YD}}{\overline{ỲF}}\implies \frac{\overline{ND}}{\overline{NE}}.\frac{\overline{ME}}{\overline{MF}}.\frac{\overline{PF}}{\overline{PD}}=\frac{\overline{CD}}{\overline{CF}}.\frac{\overline{XF}}{\overline{XE}}.\frac{\overline{ZE}}{\overline{ZD}}.\frac{\overline{BF}}{\overline{BE}}.\frac{\overline{AE}}{\overline{AD}}.\frac{\overline{YD}}{\overline{YF}}$.

Mặt khác, áp dụng định lý Menelaus cho tam giác $DEF$ với các cát tuyến $ABC,XYZ$ ta có: $\frac{\overline{AE}}{\overline{AD}}.\frac{\overline{CD}}{\overline{CF}}.\frac{\overline{BF}}{\overline{BE}}=\frac{\overline{XF}}{\overline{XE}}.\frac{\overline{ZE}}{\overline{ZD}}.\frac{\overline{YD}}{\overline{YF}}=1$. Suy ra: $\frac{\overline{ND}}{\overline{NE}}.\frac{\overline{ME}}{\overline{MF}}.\frac{\overline{PF}}{\overline{PD}}=1$.Do đó, $M,N,P$ thẳng hàng.


Bài viết đã được chỉnh sửa nội dung bởi tritanngo99: 08-12-2018 - 22:17


#89
tritanngo99

tritanngo99

    Đại úy

  • Điều hành viên THPT
  • 1644 Bài viết

Bài 157: Tìm tất cả các số nguyên tố $x$ sao cho $[\sqrt{1}]+[\sqrt{2}]+...+[\sqrt{x^2-1}]$ là số nguyên tố.

Bài 158: Xét bàn cờ vua $8x8$. Chứng minh rằng nếu xuất phát từ một ô góc, con mã không thể đi qua tất cả các ô của bàn cờ, mỗi ô một lần và kết thúc ở ô góc đối diện với ô góc nó xuất phát.

Bài 159: [Bất đẳng thức Ptolemy] Cho tứ giác lồi $ABCD$ bất kỳ, chứng minh rằng: $AB.CD+BC.AD\ge AC.BD$. Đẳng thức xảy ra khi và chỉ khi $ABCD$ là tứ giác nội tiếp.

Bài 160: [Định lý Pascal] Cho $6$ điểm bất kì $A,B,C,D,E,F$ cùng nằm trên một đường tròn. Gọi $G,H,K$ theo thứ tự là giao điểm của cặp đường thẳng $(AB,DE),(BC,EF),(CD,FA)$. Khi đó $3$ điểm $G,H,K$ thẳng hàng.


Bài viết đã được chỉnh sửa nội dung bởi tritanngo99: 09-12-2018 - 20:27


#90
tritanngo99

tritanngo99

    Đại úy

  • Điều hành viên THPT
  • 1644 Bài viết

Lời giải bải 157: Nhận xét: $[\sqrt{n^2}]=[\sqrt{n^2+1}]=...=[\sqrt{(n+1)^2-1}]=n,n\in \mathbb{N}$.

Đặt $S_n=[\sqrt{n^2}]+[\sqrt{n^2+1}]+...+[\sqrt{(n+1)^2-1}]=n(2n+1)=2n^2+n$.

Do đó: $y=[\sqrt{1}]+[\sqrt{2}]+...+[\sqrt{x^2-1}]=S_1+S_2+...+S_{x-1}=\frac{x(4x^2+3x-1)}{6}$.

Nên $6y=x(4x^2-3x-1)$, suy ra $6y\vdots x$, mà $x,y$ là các số nguyên tố suy ra $x\in \left\{2;3;y\right\}$.

Nếu $x=2$ thì $y=3$( thỏa mãn);

Nếu $x=3$ thì $y=13$( thỏa mãn);

Nếu $x=y$ thì $y=-1$ hoặc $y=\frac{7}{4}$( loại).

Vậy bài toán có hai nghiệm là $x=2$ hoặc $x=3$.

Lời giải bài 158: Mỗi lần đi, con mã chuyển sang ô màu khác với ô nó đang đứng.

Để đi qua tất cả các ô của bàn cờ, mỗi ô một lần thì con mã phải đi hết $63$ bước. Do đó, ô mà nó kết thúc hành trình khác màu với ô ban đầu nó xuất phát. Đó không thể là ô ở góc đối diện.

Lời giải bài 159: Trong tứ giác $ABCD$, lấy điểm $E$ sao cho $\angle{EAB}=\angle{DAC};\angle{EBA}=\angle{ACD}$.

$\implies \angle{BAC}=\angle{EAD}$. Khi đó $\triangle{ABE}\sim \triangle{ACD}$ nên $\frac{AB}{AC}=\frac{BE}{CD}=\frac{AE}{AD}\implies AB.CD=AC.BE$ và $\triangle{AED}\sim \triangle{ABC}$. Suy ra $\frac{AD}{AC}=\frac{AD}{BC}\implies AD.BC=AC.ED$.

Do đó $AB.CD+AD.BC=AC(BE+ED)\ge AC.BD$.

Đẳng thức xảy ra khi và chỉ khi $E\in BD\iff \angle{ABD}=\angle{ABE}=\angle{ACD}\iff$ $ABCD$ là tứ giác nội tiếp.

Từ đó suy ra định lý Ptolemy: Tứ giác lồi $ABCD$ là tứ giác nội tiếp $\iff AB.CD+BC.AD=AC.BD$.

Lời giải bài 160: 

 Sử dụng góc định hướng của $2$ đường thẳng.

Gọi $I$ là giao điểm thứ $2$ của $2$ đường tròn $(DBG)$ và $(DFK)$. Ta có:

$(IB,IF)\equiv (IB,ID)+(ID,IF)\equiv (GB,GD)+(KD,KF)(\text{ mod }\pi)$.

Mặt khác:

$(KD,KF)\equiv \frac{1}{2}((OC,OA)-(OF,OD))\equiv ((OC,OB)+(OB,OA)-(OF,OE)-(OE,OD))(\text{ mod }\pi)$.

$(GB,GD)\equiv \frac{1}{2}((OA,OE)-(OD,OB))\equiv ((OA,OF)+(OF,OE)-(OD,OC)-(OC,OB))(\text{ mod }\pi)$.

$(HB,HF)\equiv \frac{1}{2}((OB,OF)-(OE,OC))(\text{ mod }\pi)$.

$\implies (HB,HF)\equiv (KD,KF)+(GB,GD)\equiv (IB,IF)(\text{ mod }\pi)\implies B,H,I,F$ đồng viên.

Lại có $(IB,IG)\equiv (DB,DG)\equiv (FB,FE)(\text{ mod }\pi)$

$4$ điểm $B,H,I,F$ đồng viên $\implies (FB,FE)\equiv (IB,IH)(\text{ mod }\pi)$.

Do đó $(IB,IG)\equiv (IB,IH)(\text{ mod }\pi )$ hay $3$ điểm $I,G,H$ thẳng hàng. Tương tự, ta có $I,H,K$ thẳng hàng, suy ra đpcm.


Bài viết đã được chỉnh sửa nội dung bởi tritanngo99: 09-12-2018 - 20:27


#91
tritanngo99

tritanngo99

    Đại úy

  • Điều hành viên THPT
  • 1644 Bài viết

Bài 161: Giải phương trình: $[x]^2-3[x]+2=0$.

Bài 162: Một con robot nhảy trong mặt phẳng tọa độ theo quy tắc sau: Xuất phát từ điểm $(x,y)$, con robot nhảy đến điểm $(x',y')$ xác định như sau: $x'=\frac{x+y}{2},y'=\frac{2xy}{x+y}$.

Chứng minh rằng, nếu ban đầu con Robot đứng ở điểm $(2009,2010)$ thì không bao giờ con Robot nhảy vào được trong đường tròn $(C)$ có tâm là gốc tọa độ $O$ và bán kính $R=2840$.

Bài 163:[Định lý Brianchon] Cho lục giác $ABCDEF$ ngoại tiếp đường tròn. Chứng minh rằng ba đường chéo lớn $AD,BE,CF$ đồng quy.

Bài 164:[Định lý Miquel] Cho tam giác $ABC$ và ba điểm $M,N,P$ lần lượt nằm trên $BC,CA,AB$. Khi đó các đường tròn ngoại tiếp các tam giác $APN,BPM,CMN$ đồng quy.



#92
tritanngo99

tritanngo99

    Đại úy

  • Điều hành viên THPT
  • 1644 Bài viết

Lời giải bài 161: Đặt $[x]=y(y\in \mathbb{Z})$, phương trình có dạng: $y^2-3y+2=0\iff y=1\text{ hoặc }y=2$.

Nếu $y=1\implies [x]=1\iff 1\le x<2$.

Nếu $y=2\implies [x]=2\iff 2\le x<3$.

Vậy tập nghiệm của phương trình là $[1;3)$.

Lời giải bài 162: Ta có: $x'.y'=xy$. Do đó bất biến là tích các tọa độ của điểm mà con Robot nhảy tới được. Giả sử con Robot đến được điểm $M(p,q)$, ta có: $OM=\sqrt{p^2+q^2}\ge \sqrt{2pq}=\sqrt{2.2009.2010}>2840$.

Vậy, con robot không thể nhảy vào trong đường tròn tâm $O$, bán kính $R=2840$.

Lời giải bài 163: Ta kí hiệu các tiếp điểm của $(O)$ trên $AB,BC,CD,DE,EF,FA$ lần lượt là $M,N,P,Q,R,S$. Xét cực và đối cực đối với $(O)$. Gọi $K,I,J$ lần lượt là giao điểm của các cặp đường thẳng $(SM,PQ),(MN,QR),(NP,RS)$. Vì $SM$ và $PQ$ là đường đối cực của $A$ và $D$ nên $AD$ là đường đối cực của $K$. Tương tự $BE$ và $FC$ lần lượt là đường đối cực của $I$ và $J$.

Dùng định lý Pascal cho lục giác nội tiếp $MNPQRS$ ta có $I,J,K$ thẳng hàng. Nên ta có các đường đối cực của $I,J,K$( lần lượt là $BE,CF,AD$) cùng đi qua cực của đường thẳng này( đường thẳng đi qua $I,J,K$) nên $AD,BE,CF$ đồng quy(đpcm). 

Lời giải bài 164: Gọi $S$ là giao điểm của $(BPM)$ và $(CMN)$. Ta có:

$(SN,SP)\equiv (SN,SM)+(SM,SP)\equiv (CN,CM)+(BM,BP)\equiv (CA,CB)+(BC,BA)\equiv (CA,BA)\equiv (AN,AP)(\text{ mod }\pi)$.

$\implies S\in (ANP)$ suy ra đpcm.


Bài viết đã được chỉnh sửa nội dung bởi tritanngo99: 11-12-2018 - 18:38


#93
tritanngo99

tritanngo99

    Đại úy

  • Điều hành viên THPT
  • 1644 Bài viết

Bài 165: Giải phương trình: $[x]+[2x]+[3x]+...+[2009x]=4036082$.

Bài 166: Viết các số $1,2,3,...,2010$ lên bảng. Thực hiện thuật toán sau: Mỗi lần xóa đi hai số $a,b$ bất kì và viết thêm số $c=|a-b|$. Chứng minh rằng số còn lại cuối cùng trên bảng là một số lẻ.

Bài 167: [Công thức Carnot] Cho $\triangle{ABC}$ nội tiếp $(O,R),r$ là bán kính nội tiếp. Kí hiệu $d_a,d_b,d_c$ theo thứ tự là khoảng cách từ $O$ đến các cạnh $BC,CA,AB$. Khi đó ta có hệ thức sau: $d_a+d_b+d_c=R+r$.

Bài 168: [Định lý Carnot]  Cho tam giác $ABC$, gọi $M,N,P$ lần lượt là các điểm trên các cạnh $BC,CA,AB;d_M,d_N,d_P$ là các đường thẳng đi qua $M,N,P$ và vuông góc với các cạnh tương ứng. Khi đó: $d_M,d_N,d_P$ đồng quy $\iff MB^2+NC^2+PA^2=MC^2+NA^2+PB^2$



#94
tritanngo99

tritanngo99

    Đại úy

  • Điều hành viên THPT
  • 1644 Bài viết

Lời giải bài 165: Nhận xét rằng: $[x]\le x<[x]+1$ suy ra $k[x]\le kx<k[x]+k$ nên $k[x]\le [kx]\le k[x]+k-1(k\in \mathbb{Z}^{+})$

Do đó thay $k=1,2,...,2009$ rồi cộng theo vế ta có:

$2019045[x]\le [x]+[2x]+...+[2009x]\le 2019045[x]+2017036$.

$\iff 2019045[x]\le 4036082\le 2019045[x]+2017036$.

Lại có: $4036082=2019045+2017037$. Do đó phương trình vô nghiệm.

Lời giải bài 166: Gọi $S(n)$ là tổng các số trên bảng sau bước thứ $n$. Ta có $S(n)$ bất biến theo môđun $2$. Mặt khác: $S(0)=1+2+3+...+2010=1005.2011$.

Nên $S(n)\equiv 1(\text{ mod }2),\forall n$.

Vậy số cuối cùng còn lại trên bảng là một số lẻ.

Lời giải bài 167: Gọi $D,E,F$ theo thứ tự là trung điểm $BC,CA,AB\implies OD,OE,OF\bot BC,CA,AB$. Áp dụng định lý Ptolemy cho tứ giác nội tiếp $AEOF$, ta có: $OA.EF=AF.OE+AE.OF\implies aR=cd_b+bd_c$. Tương tự: $bR=ad_c+cd_a;cR=ad_b+bd_a$. Suy ra $R(a+b+c)=a(d_b+d_c)+b(d_c+d_a)+c(d_a+d_b)$.

Mặt khác: $r(a+b+c)=2S_{ABC}=ad_a+bd_b+cd_c$.

Do đó: $(a+b+c)(R+r)=(a+b+c)(d_a+d_b+d_c)\implies d_a+d_b+d_c=R+r$(đpcm).

Nếu tam giác $ABC$ có $\angle{A}>90^0$ thì ta có $-d_a+d_b+d_c=R+r$.

Chú ý rằng định lý Carnot tương đương với hệ thức quen thuộc sau: $cosA+cosB+cosC=1+\frac{r}{R}$.

Lời giải bài 168: Chứng minh:

Thuận: Gọi $O$ là giao điểm của $3$ đường thẳng. Ta có:

$MB^2+NC^2+PA^2=MC^2+NA^2+PB^2$.

$\iff MB^2+OM^2+NC^2+ON^2+PA^2+OP^2=MC^2+OM^2+NA^2+ON^2+PB^2+OP^2$.

$\iff OB^2+OC^2+OA^2=OC^2+OA^2+OB^2$.

Đẳng thức này luôn đúng nên ta có điều phải chứng minh.

Đảo: Gọi $O$ là giao điểm của $d_M,d_N$. Qua $O$, hạ đường vuông góc xuống $AB$ tại $P'$. Áp dụng định lý thuận, ta có: $P'A^2-P'B^2=PA^2-PB^2\implies P\equiv P'\implies $ đpcm.


Bài viết đã được chỉnh sửa nội dung bởi tritanngo99: 17-12-2018 - 09:41


#95
tritanngo99

tritanngo99

    Đại úy

  • Điều hành viên THPT
  • 1644 Bài viết

Bài 169: Chứng minh rằng từ $n+1$ số dương khác nhau nhỏ hơn $2n$, có thể chọn được ba số sao cho tổng hai số trong chúng bằng số thứ ba.

Bài 170: Một hình tròn được chia thành $2010$ hình quạt. Trong mỗi hình quạt có một viên bi. Thực hiện trò chơi sau: Mỗi lần cho phép lấy ra hai viên bi trong hai hình quạt nào đó và chuyển chúng sang các ô bên cạnh nhưng theo hai chiều ngược nhau. Hỏi sau một số lần có thể chuyển hết các viên bi vào một hình quạt được không?

Bài 171: [Khái niệm trực giao] Cho tam giác $ABC$ và tam giác $A_1B_1C_1$ như hình vẽ. Nếu các đường thẳng qua $A_1,B_1,C_1$ và vuông góc với $BC,CA,AB$ đồng quy thì các đường thẳng qua $A,B,C$ và vuông góc $B_1C_1,C_1A_1,A_1B_1$ đồng quy và ngược lại. Khi đó $2$ tam giác $ABC$ và $A_1B_1C_1$ được gọi là $2$ tam giác trực giao.

121.jpg

Bài 172: [Định lý Brocard] Cho tứ giác lồi $ABCD$ nội tiếp đường tròn tâm $O$. $AD$ giao $BC$ tại $M$, $AB$ giao $CD$ tại $N$, $AC$ giao $BD$ tại $I$. Chứng minh rằng $O$ là trực tâm của tam giác $MIN$.


Bài viết đã được chỉnh sửa nội dung bởi tritanngo99: 17-12-2018 - 09:56


#96
tritanngo99

tritanngo99

    Đại úy

  • Điều hành viên THPT
  • 1644 Bài viết

Lời giải bài 169: Ký hiệu $0<a_1<a_2<...<a_{n+1}$ là những số đã cho. Chúng ta xét các hiệu số $a_2-a_1,a_3-a_1,...,a_{n+1}-a_1$ và các số $a_2,a_3,...,a_{n+1}$. Vì tất cả các số này đều nhỏ hơn $2n$ nên các số trên chỉ nằm trong khoảng $1,2,...,2n-1$. Như vậy chúng ta sẽ tìm một số ở nhóm thứ nhất bằng một số ở nhóm thứ $2:a_k-a_1=a_l$. Suy ra $a_k=a_1+a_l$

Lời giải bài 170: Tô màu các hình quạt bởi hai màu đen, trắng như hình vẽ: 

sao cho hai hình quạt kề nhau thì khác màu. Gọi $S(n)$ và $T(n)$ tương ứng là số viên bi trong các hình quạt màu đen và số viên bi trong các hình quạt màu trằng sau bước chuyển bi thứ $n$. Ta có $S(n)$ và $T(n)$ bất biến theo modulo $2$. Do $S(0)=T(0)=1005$ nên $S(n)$ và $T(n)$ lẻ với mọi $n$. Do đó không thể có trạng thái mà tất cả các viên bi ở trong cùng một hình quạt.

Lời giải bài 171: Gọi $M,N,P,M_1,N_1,P_1$ là chân các đường vuông góc như hình vẽ. Áp dụng định lý Carnot, ta có: $AM_1,BN_1,CP_1$ đồng quy.

$\iff (M_1C_1^2-M_1B_1^2)+(P_1B_1^2-P_1A_1^2)+(N_1A_1^2-N_1C_1^2)=0$.

$\iff (AC_1^2-AB_1^2)+(CB_1^2-CA_1^2)+(BA_1^2-BC_1^2)=0$

$\iff (A_1B^2-A_1C^2)+(B_1C^2-B_1A^2)+(C_1A^2-C_1B^2)=0$

$\iff (MB^2-MC^2)+(NC^2-NA^2)+(PA^2-PB^2)=0$

$\iff A_1M,B_1M,C_1P$ đồng quy. (đpcm).

Lời giải bài 172: Gọi $H$ là giao điểm thứ $2$ của hai đường tròn ngoại tiếp các tam giác $AID,BIC$. Xét tứ giác $DOHC$, ta có: $\angle{DHC}=360^0-\angle{DHI}-\angle{CHI}=\angle{DAC}+\angle{DBC}=\angle{DOC}$.

Từ đó suy ra tứ giác $DOHC$ nội tiếp. Tương tự, ta suy ra tứ giác $AOHB$ nội tiếp. Mặt khác $\overline{MA}.\overline{MD}=\overline{MB}.\overline{MC},$ suy ra $M$ nằm trên trục đẳng phương của hai đường tròn $(AIHD),(BIHC)\implies M,I,H$ thẳng hàng.

Ta có: $\angle{IHO}=\angle{IHD}-\angle{OHD}=\angle{ADC}+\angle{ACD}-\angle{OCD}=\angle{ADC}+\angle{OCA}=90^0$.

Từ đó suy ra $IM\bot ON$. Tương tự, ta có $IN\bot OM$. Suy ra đpcm


Bài viết đã được chỉnh sửa nội dung bởi tritanngo99: 18-12-2018 - 20:52


#97
tritanngo99

tritanngo99

    Đại úy

  • Điều hành viên THPT
  • 1644 Bài viết

Bài 173: Cho $p$ là số nguyên tố lớn hơn $5$. Chứng minh rằng tồn tại một số có dạng $111...111$ mà chia hết cho $p$.

Bài 174: Một dãy gồm có $19$ phòng. Ban đầu mỗi phòng có một người. Sau đó, cứ mỗi ngày có hai người nào đó chuyển sang hai phòng bên cạnh nhưng theo hai chiều ngược nhau. Hỏi sau một số ngày có hay không trường hợp mà 

(a) Không có ai ở phòng có thứ tự chẵn.

(b) Có $10$ người ở phòng cuối.

Bài 175: [Định lý Ở-le về khoảng cách tâm giữa $2$ đường tròn nội, ngoại tiếp của tam giác]

Cho $\triangle{ABC}$ nội tiếp $(O,R)$, ngoại tiếp $(I,r)$. Khi đó $OI^2=R^2-2Rr$.

Bài 176: [Định lý Ở-le về khoảng cách tâm giữa $2$ đường tròn nội, ngoại tiếp của tứ giác( Định lý Fuss)]

Cho tứ giác $ABCD$ vừa nội tiếp $(O,R)$, vừa ngoại tiếp $(I,r)$. Đặt $OI=d$. Khi đó ta có hệ thức $\frac{1}{(R-d)^2}+\frac{1}{(R+d)^2}=\frac{1}{r^2}$ 



#98
tritanngo99

tritanngo99

    Đại úy

  • Điều hành viên THPT
  • 1644 Bài viết

Lời giải bài 173: Ta xét dãy số $1,11,111,...,\underbrace{111...1}_{p\text{ chữ số}}$. Nếu trong dãy trên không có số nào chia hết chia hết cho $p$, thì ta cho tương ứng mỗi số với số dư của phép chia. Tập hợp các số dư chỉ có $1,2,...,p-1$ gồm $p-1$ phần tử( vì $0$ không thể có trong tập này ). Nhưng vì chúng ta có $p$ số ở dạng trên, nên theo nguyên lý Dirichlet tồn tại hai số có cùng số dư. Giả sử các số đó là $\underbrace{111...1}_{m\text{ chữ số}}$ và $\underbrace{111...1}_{n\text{ chữ số}}$ với $m>n$. Khi đó $1\le n<m\le p$. Vậy $\underbrace{111...1}_{m\text{ chữ số}}-\underbrace{111...1}_{n\text{ chữ số}}=\underbrace{111...000}_{m-n\text{ chữ số }1,n\text{ chữ số }0}=\underbrace{111...1}_{m-n\text{ chữ số }}.10^n$

Tích này chia hết cho $p$ vì $(p,10)=1$, suy ra $\underbrace{111...1}_{m-n\text{ chữ số }1}$

Lời giải bài 174: Đánh số các phòng theo thứ tự $1,2,3,...,19$.

Ta đeo cho mỗi vị khách một thẻ ghi số phòng mình đang ở. Gọi $S(n)$ là tổng các số ghi trên thẻ của tất cả các vị khách ở ngày thứ $n$. Dễ thấy $S(n)$ là bất biến. Do đó, ta có: $S(n)=S(1)=1+2+...+19=190,\forall n\ge 1$.

a)  Vì có lẻ người nên nếu không ai ở phòng có thứ tự chẵn thì $S(n)$là một số lẻ, vô lý.

b) Nếu có $10$ người ở phòng cuối (phòng số $19$) thì $S(n)>19.10=190$, vô lý.

Lời giải bài 175: Gọi $D,E$ theo thứ tự là trung điểm các cung nhỏ $BC$ và $AC$ thì $OD\bot BC;\angle{BAD}=\frac{\angle{A}}{2}$. Gọi $H$ là chân đường vuông góc kẻ từ $I$ xuống $OD$. $J$ là trung điểm $BC$. Theo một kết quả quen biết, ta có: $ID=BC=2R.sin\frac{A}{2}$.

Trong $\triangle{OID}$, có $OI^2=ID^2+OD^2-2\vec{ID}.\vec{OD}=4R^2sin^2\frac{A}{2}+R^2-2\vec{DO}.\vec{DH}$ (công thức hình chiếu).

Mặt khác, $\vec{DO}.\vec{DH}=\vec{DP}(\vec{DJ}+\vec{JH})=R(BD.sin\frac{A}{2}+r)=2R^2sin^2\frac{A}{2}+Rr$. Từ đó suy ra đpcm.

Lời giải bài 176: Gọi tiếp điểm của $(I)$ trên $AB,BC,CD,DA$ lần lượt là $M,N,P,Q$. $BI,DI$ cắt $(O)$ lần lượt ở $E,F$. Ta thấy:

$(DE,DF)\equiv (DE,DC)+(DC,DF)\equiv (BE,BC)+(DC,DF)\equiv \frac{(BA,BC)+(DC,DA)}{2}\equiv \frac{\pi}{2}(\text{ mod }\pi)$.

Do đó $E,O,F$ thẳng hàng, nên $O$ là trung điểm của $EF$. Theo công thức đường trung tuyến trong tam giác $IEF$, ta có: $d^2=\frac{IE^2+IF^2}{2}-\frac{EF^2}{4}=\frac{IE^2+IF^2}{2}-R^2$. Suy ra:

$\frac{1}{(R-d)^2}+\frac{1}{(R+d)^2}=\frac{2(R^2+d^2)}{(R^2-d^2)^2}=\frac{IE^2+IF^2}{(P_{I/(O)})^2}=\frac{IE^2}{P_{I/(O)})^2}+\frac{IF^2}{P_{I/(O)})^2}=\frac{IE^2}{(IE.IB)^2}+\frac{IF^2}{(IF.ID)^2}=\frac{1}{IB^2}+\frac{1}{ID^2}=\frac{sin^2\frac{B}{2}}{IM^2}+\frac{sin^2\frac{D}{2}}{IP^2}=\frac{1}{r^2}$. 


Bài viết đã được chỉnh sửa nội dung bởi tritanngo99: 20-12-2018 - 20:20


#99
tritanngo99

tritanngo99

    Đại úy

  • Điều hành viên THPT
  • 1644 Bài viết

Bài 177: Cho $M$ là tập hợp bất kì gồm $10$ số tự nhiên, mỗi số không lớn hơn $100$. Chứng minh rằng tồn tại hai tập con của $M$ mà tổng của các phần tử trong chúng bằng nhau.

Bài 178: Ở $6$ đỉnh của một lục giác lồi có ghi $6$ số chẵn liên tiếp theo chiều kim đồng hồ. Ta thay đổi các số như sau: Mỗi lần lấy ra một cạnh và cộng hai số trên cạnh đó với cùng một số nguyên nào đó. Hỏi sau một số lần thay đổi như thế thì $6$ số mới ở các đỉnh của lục giác có thể bằng nhau không?

Bài 179: [Định lý Casey] (Định lý Ptolemy mở rộng) Cho tứ giác $ABCD$ nội tiếp $(O;R)$. Đặt các đường tròn $\alpha,\beta,\gamma,\delta$ là các đường tròn tiếp xúc với $(O)$ tại các đỉnh $A,B,C,D$. Đặt $t_{\alpha,\beta}$ là độ dài đoạn tiếp tuyến chung của hai đường tròn $\alpha,\beta$. Trong đó $t_{\alpha,\beta}$ là độ dài tiếp tuyến chung ngoài nếu hai đường tròn $\alpha,\beta$ cùng tiếp xúc trong hoặc cùng tiếp xúc ngoài với $(O)$, và là độ dài đoạn tiếp xúc trong với trường hợp còn lại. Các đoạn $t_{\alpha\gamma}, t_{\beta\gamma},..$ được xác định tương tự. Khi đó ta có: $t_{\alpha\beta}t_{\gamma\delta}+t_{\beta\gamma}t_{\alpha\delta}=t_{\alpha\gamma}t_{\beta\delta}$.

Bài 180: (Định lý Stewart) Cho $3$ điểm $A,B,C$ thẳng hàng và điểm $M$ bất kì. Ta có hệ thức sau: $MA^2.\overline{BC}+MB^2.\overline{CA}+MC^2.\overline{AB}+\overline{BC}.\overline{CA}.\overline{AB}=0$.


Bài viết đã được chỉnh sửa nội dung bởi tritanngo99: 22-12-2018 - 06:30


#100
tritanngo99

tritanngo99

    Đại úy

  • Điều hành viên THPT
  • 1644 Bài viết

Lời giải bài 177: Có thể chứng minh nếu tồn tại hai tập thỏa mãn kết luận của bài toán, thì ta có thể chọn được hai tập con có cùng tính chất ấy nhưng không giao nhau. Thật vậy, cho $X,Y$ là hai tập con của $M$ có tổng các phần tử bằng nhau. Chúng ta ký hiệu $X_1$ gồm các phần tử của $X$ mà không thuộc $Y$. Tương tự như vậy $Y_1$ gồm các phần tử của $Y$ mà không thuộc $X$. Rõ ràng $X_1$ và $Y_1$ có tổng các phần tử bằng nhau mà không giao nhau. Gọi $A$ là tập hợp mọi tập hợp con không rỗng của $M$. Số lượng phần tử của $A$ là $2^{10}-1=1023$. Chúng ta xét tổng $S$ các phần tử của một tập hợp con như vậy, rõ ràng $S\le 91+92+...+100<10.100=1000$. Như vậy tồn tại không quá $1000$ tổng khác nhau. Ký hiệu $B$ là tập hợp tất cả các tổng như vậy. Do đó số lượng phần tử của $B$ nhỏ hơn $1000$ và nhỏ hơn số lượng phần tử của $A$. Đặt tương ứng mỗi phần tử của tập hợp $A$ với tổng các phần tử của nó. Ta thấy rằng có thể áp dụng nguyên lý DDirrichlet ở đây. Suy ra tồn tại ít nhất hai tập hợp con khác nhau có cùng một tổng các phần tử.

Lời giải bài 178: Giả sử lục giác đã cho là $A_1A_2A_3A_4A_5A_6$ và $6$ số ghi ở các đỉnh $A_1,A_2,A_3,A_4,A_5,A_6$ ban đầu tương ứng là $a,a+2,a+4,a+6,a+8,a+10$ với $a\in \mathbb{Z}$.

Gọi $S(n)$ và $T(n)$ tương ứng là tổng các số ở các đỉnh $A_1,A_3,A_5$ và $A_2,A_4,A_6$ sau bước thứ $n$. Ta có $S(n)-T(n)$ là bất biến. Từ đó suy ra không nhận được trạng thái mà $6$ số ở $6$ đỉnh của lục giác bằng nhau.

Lời giải bài 179: Ta chứng minh cho trường hợp $\alpha,\beta,\gamma,\delta$ cùng tiếp xúc ngoài với $(O)$, các trường hợp khác chứng minh tương tự. Gọi tâm các đường tròn trên là $A',B',C',D'$ và bán kính lần lượt là $x,y,z,t$. Đặt độ dài các đoạn thẳng như hình vẽ và $AC=m,BD=n$. Theo định lý Pytagore: $t_{\alpha\beta}^2=A'B'^2-(x-y)^2$. Áp dụng định lý cosin, ta có:

$A'B'^2=(R+x)^2+(R+y)^2-2(R+x)(R+y)cos\angle{A'OB'}=(R+x)^2+(R+y)^2-2(R+x)(R+y)(1-\frac{a^2}{2R^2})$.

$=(R+x)^2+(R+y)^2-2(R+x)(R+y)+(R+x)(R+y)\frac{a^2}{R^2}=(x-y)^2+(R+x)(R+y)\frac{a^2}{R^2}$

$\implies t_{\alpha\beta}=\frac{a}{R}\sqrt{(R+x)(R+y)}$.

Tương tự với các đoạn thẳng còn lại, ta có $t_{\alpha\beta}t_{\gamma\delta}+t_{\beta\gamma}t_{\alpha\delta}=t_{\alpha\gamma}t_{\beta\delta}\iff ac+bd=mn$( luôn đúng theo định lý Ptolemy).

Cho $x=y=z=t=0$, ta có định lý Ptolemy.

11.png

Lời giải bài 180: Qua $M$, hạ $MH\bot ABC$, ta có:

$MA^2\overline{BC}+MB^2\overline{CA}+MC^2\overline{AB}+\overline{BC}.\overline{CA}.\overline{AB}$.

$=MH^2(\overline{BC}+\overline{CA}+\overline{AB})+HA^2\overline{BC}+HB^2\overline{CA}+HC^2\overline{AB}+\overline{BC}.\overline{CA}.\overline{AB}$

$=HA^2\overline{BC}+HB^2\overline{CA}+HC^2\overline{AB}+\overline{BC}.\overline{CA}.\overline{AB}$

$=HA^2(\overline{HC}-\overline{HB})+HB^2(\overline{HA}-\overline{HC})+HC^2(\overline{HB}-\overline{HA})+(\overline{HC}-\overline{HB})(\overline{HA}-\overline{HC})(\overline{HB}-\overline{HA})=0$


Bài viết đã được chỉnh sửa nội dung bởi tritanngo99: 23-12-2018 - 18:15





2 người đang xem chủ đề

0 thành viên, 2 khách, 0 thành viên ẩn danh